Clean up unused/not to be proven.

finite-set-exercises
Joshua Potter 2023-05-03 13:09:41 -06:00
parent fcbd510dbe
commit a8f12f2ec0
2 changed files with 15 additions and 154 deletions

View File

@ -1,98 +0,0 @@
/-
Exercises 1.7
-/
import OneVariableCalculus.Real.Geometry.Area
import OneVariableCalculus.Real.Geometry.Rectangle
open Real
open Real.Geometry.Area
-- ========================================
-- The properties of area in this set of exercises are to be deduced from the
-- axioms for area stated in the foregoing section.
-- ========================================
-- ----------------------------------------
-- Exercise 1
--
-- Prove that each of the following sets is measurable and has zero area:
-- (a) A set consisting of a single point.
-- (b) A set consisting of a finite number of points in a plane.
-- (c) The union of a finite collection of line segments in a plane.
-- ----------------------------------------
example (p : Point)
: p.set_def ∈ 𝓜
∧ ((h : p.set_def ∈ 𝓜) → area h = 0) :=
sorry
example (S : Set Point) (hf : Set.Finite S)
: ( p ∈ S, p.set_def) ∈ 𝓜
∧ ((h : ( p ∈ S, p.set_def) ∈ 𝓜) → area h = 0) :=
sorry
example (S : Set LineSegment) (hf : Set.Finite S)
: ( s ∈ S, s.set_def) ∈ 𝓜
∧ ((h : ( s ∈ S, s.set_def) ∈ 𝓜) → area h = 0) :=
sorry
-- ----------------------------------------
-- Exercise 2
--
-- Every right triangular region is measurable because it can be obtained as the
-- intersection of two rectangles. Prove that every triangular region is
-- measurable and that its area is one half the product of its base and
-- altitude.
-- ----------------------------------------
-- # TODO
-- ----------------------------------------
-- Exercise 3
--
-- Prove that every trapezoid and every parallelogram is measurable and derive
-- the usual formulas for their areas.
-- ----------------------------------------
-- # TODO
-- ----------------------------------------
-- Exercise 4
--
-- A point `(x, y)` in the plane is called a *lattice point* if both coordinates
-- `x` and `y` are integers. Let `P` be a polygon whose vertices are lattice
-- points inside the polygon and `B` denotes the number on the boundary.
--
-- (a) Prove that the formula is valid for rectangles with sides parallel to the
-- coordinate axes.
-- (b) Prove that the formula is valid for right triangles and parallelograms.
-- (c) Use induction on the number of edges to construct a proof for general
-- polygons.
-- ----------------------------------------
-- # TODO
-- ----------------------------------------
-- Exercise 5
--
-- Prove that a triangle whose vertices are lattice points cannot be
-- equilateral.
--
-- [Hint: Assume there is such a triangle and compute its area in two ways,
-- using exercises 2 and 4.]
-- ----------------------------------------
-- # TODO
-- ----------------------------------------
-- Exercise 6
--
-- Let `A = {1, 2, 3, 4, 5}`, and let `𝓜` denote the class of all subsets of
-- `A`. (There are `32` altogether, counting `A` itself and the empty set `∅`.)
-- For each set `S` in `𝓜`, let `n(S)` denote the number of distinct elements in
-- `S`. If `S = {1, 2, 3, 4}` and `T = {3, 4, 5}`, compute `n(S T)`,
-- `n(S ∩ T)`, `n(S - T)`, and `n(T - S)`. Prove that the set function `n`
-- satisfies the first three axioms for area.
-- ----------------------------------------
-- # TODO

View File

@ -19,7 +19,7 @@ import OneVariableCalculus.Apostol.Chapter_I_3
-- at least one real `z` satisfying `x < z < y`. -- at least one real `z` satisfying `x < z < y`.
-- ======================================== -- ========================================
example (x y : ) (h : x < y) : ∃ z, x < z ∧ z < y := by theorem exercise1 (x y : ) (h : x < y) : ∃ z, x < z ∧ z < y := by
have ⟨z, hz⟩ := exists_pos_add_of_lt' h have ⟨z, hz⟩ := exists_pos_add_of_lt' h
refine ⟨x + z / 2, ⟨?_, ?_⟩⟩ refine ⟨x + z / 2, ⟨?_, ?_⟩⟩
· have hz' : z / 2 > 0 := by · have hz' : z / 2 > 0 := by
@ -38,7 +38,7 @@ example (x y : ) (h : x < y) : ∃ z, x < z ∧ z < y := by
-- such that `m < x < n`. -- such that `m < x < n`.
-- ======================================== -- ========================================
example (x : ) : ∃ m n : , m < x ∧ x < n := by theorem exercise2 (x : ) : ∃ m n : , m < x ∧ x < n := by
refine ⟨x - 1, ⟨x + 1, ⟨?_, ?_⟩⟩⟩ <;> norm_num refine ⟨x - 1, ⟨x + 1, ⟨?_, ?_⟩⟩⟩ <;> norm_num
-- ======================================== -- ========================================
@ -47,7 +47,7 @@ example (x : ) : ∃ m n : , m < x ∧ x < n := by
-- If `x > 0`, prove that there is a positive integer `n` such that `1 / n < x`. -- If `x > 0`, prove that there is a positive integer `n` such that `1 / n < x`.
-- ======================================== -- ========================================
example (x : ) (h : x > 0) : ∃ n : +, 1 / n < x := by theorem exercise3 (x : ) (h : x > 0) : ∃ n : +, 1 / n < x := by
have ⟨n, hn⟩ := @Real.exists_pnat_mul_self_geq_of_pos x 1 h have ⟨n, hn⟩ := @Real.exists_pnat_mul_self_geq_of_pos x 1 h
refine ⟨n, ?_⟩ refine ⟨n, ?_⟩
have hr := mul_lt_mul_of_pos_right hn (show 0 < 1 / ↑↑n by norm_num) have hr := mul_lt_mul_of_pos_right hn (show 0 < 1 / ↑↑n by norm_num)
@ -63,7 +63,7 @@ example (x : ) (h : x > 0) : ∃ n : +, 1 / n < x := by
-- `⌊5 / 2⌋ = 2`, `⌊-8/3⌋ = -3`. -- `⌊5 / 2⌋ = 2`, `⌊-8/3⌋ = -3`.
-- ======================================== -- ========================================
example (x : ) : ∃! n : , n ≤ x ∧ x < n + 1 := by theorem exercise4 (x : ) : ∃! n : , n ≤ x ∧ x < n + 1 := by
let n := Int.floor x let n := Int.floor x
refine ⟨n, ⟨?_, ?_⟩⟩ refine ⟨n, ⟨?_, ?_⟩⟩
· exact ⟨Int.floor_le x, Int.lt_floor_add_one x⟩ · exact ⟨Int.floor_le x, Int.lt_floor_add_one x⟩
@ -78,7 +78,7 @@ example (x : ) : ∃! n : , n ≤ x ∧ x < n + 1 := by
-- `n` which satisfies `x ≤ n < x + 1`. -- `n` which satisfies `x ≤ n < x + 1`.
-- ======================================== -- ========================================
example (x : ) : ∃! n : , x ≤ n ∧ n < x + 1 := by theorem exercise5 (x : ) : ∃! n : , x ≤ n ∧ n < x + 1 := by
let n := Int.ceil x let n := Int.ceil x
refine ⟨n, ⟨?_, ?_⟩⟩ refine ⟨n, ⟨?_, ?_⟩⟩
· exact ⟨Int.le_ceil x, Int.ceil_lt_add_one x⟩ · exact ⟨Int.le_ceil x, Int.ceil_lt_add_one x⟩
@ -101,8 +101,7 @@ example (x : ) : ∃! n : , x ≤ n ∧ n < x + 1 := by
-- are *dense* in the real-number system. -- are *dense* in the real-number system.
-- ======================================== -- ========================================
example (x y : ) (h : x < y) : ∃ r : , x < r ∧ r < y := by -- # TODO
sorry
-- ======================================== -- ========================================
-- Exercise 7 -- Exercise 7
@ -111,25 +110,7 @@ example (x y : ) (h : x < y) : ∃ r : , x < r ∧ r < y := by
-- `xy`, `x / y`, and `y / x` are all irrational. -- `xy`, `x / y`, and `y / x` are all irrational.
-- ======================================== -- ========================================
example (x : ) (hx : x ≠ 0) (y : ) (hy : irrational y) -- # TODO
: irrational (x + y) :=
sorry
example (x : ) (hx : x ≠ 0) (y : ) (hy : irrational y)
: irrational (x - y) :=
sorry
example (x : ) (hx : x ≠ 0) (y : ) (hy : irrational y)
: irrational (x * y) :=
sorry
example (x : ) (hx : x ≠ 0) (y : ) (hy : irrational y)
: y ≠ 0 → irrational (x / y) :=
sorry
example (x : ) (hx : x ≠ 0) (y : ) (hy : irrational y)
: irrational (y / x) :=
sorry
-- ======================================== -- ========================================
-- Exercise 8 -- Exercise 8
@ -137,10 +118,7 @@ example (x : ) (hx : x ≠ 0) (y : ) (hy : irrational y)
-- Is the sum or product of two irrational numbers always irrational? -- Is the sum or product of two irrational numbers always irrational?
-- ======================================== -- ========================================
-- No. Here is a counterexample. -- # TODO
example (hx : x = Real.sqrt 2): irrational x ∧ rational (x * x) := by
sorry
-- ======================================== -- ========================================
-- Exercise 9 -- Exercise 9
@ -150,8 +128,7 @@ example (hx : x = Real.sqrt 2): irrational x ∧ rational (x * x) := by
-- infinitely many. -- infinitely many.
-- ======================================== -- ========================================
example (x y : ) (h : x < y) : ∃ z : , irrational z ∧ x < z ∧ z < y := by -- # TODO
sorry
-- ======================================== -- ========================================
-- Exercise 10 -- Exercise 10
@ -171,34 +148,27 @@ def is_odd (n : ) := is_even (n + 1)
-- (a) An integer cannot be both even and odd. -- (a) An integer cannot be both even and odd.
-- ---------------------------------------- -- ----------------------------------------
example (n : ) : is_even n = ¬ is_odd n := sorry -- # TODO
-- ---------------------------------------- -- ----------------------------------------
-- (b) Every integer is either even or odd. -- (b) Every integer is either even or odd.
-- ---------------------------------------- -- ----------------------------------------
example (n : ) : is_even n is_odd n := sorry -- # TODO
-- ---------------------------------------- -- ----------------------------------------
-- (c) The sum or product of two even integers is even. What can you say about -- (c) The sum or product of two even integers is even. What can you say about
-- the sum or product of two odd integers? -- the sum or product of two odd integers?
-- ---------------------------------------- -- ----------------------------------------
example (m n : ) : is_even m ∧ is_even n → is_even (m * n) := sorry -- # TODO
example (m n : ) :
(∃ m n : , is_odd m ∧ is_odd n ∧ is_even (m * n)) ∧
(∃ m n : , is_odd m ∧ is_odd n ∧ is_odd (m * n)) :=
sorry
-- ---------------------------------------- -- ----------------------------------------
-- (d) If `n²` is even, so is `n`. If `a² = 2b²`, where `a` and `b` are -- (d) If `n²` is even, so is `n`. If `a² = 2b²`, where `a` and `b` are
-- integers, then both `a` and `b` are even. -- integers, then both `a` and `b` are even.
-- ---------------------------------------- -- ----------------------------------------
example (n : ) (h : is_even (n ^ 2)) : is_even n := sorry -- # TODO
example (a b : ) (h : a ^ 2 = 2 * b ^ 2) : is_even a ∧ is_even b := sorry
-- ======================================== -- ========================================
-- Exercise 11 -- Exercise 11
@ -210,7 +180,7 @@ example (a b : ) (h : a ^ 2 = 2 * b ^ 2) : is_even a ∧ is_even b := sorry
-- contradiction.] -- contradiction.]
-- ======================================== -- ========================================
example : ¬ ∃ n : , rational n → n ^ 2 = 2 := sorry -- # TODO
-- ======================================== -- ========================================
-- Exercise 12 -- Exercise 12
@ -222,15 +192,4 @@ example : ¬ ∃ n : , rational n → n ^ 2 = 2 := sorry
-- least-upper-bound axiom. -- least-upper-bound axiom.
-- ======================================== -- ========================================
/-- -- # TODO
Shows the set of rational numbers satisfies the Archimedean property.
-/
theorem exists_pnat_mul_self_geq_of_pos {x y : }
: x > 0 → ∃ n : +, n * x > y := sorry
/--
Show the Archimedean property does not imply the least-upper-bound axiom.
-/
example (S : Set ) (hne : S.Nonempty) (hbdd : BddAbove S)
: ¬ ∃ x, IsLUB S x :=
sorry